User avatar
 
LSAT-Chang
Thanks Received: 38
Atticus Finch
Atticus Finch
 
Posts: 479
Joined: June 03rd, 2011
 
 
trophy
Most Thankful
trophy
First Responder
 

Q13 - Philosopher: Some of the most

by LSAT-Chang Thu Sep 15, 2011 9:41 pm

I was very confused by this question... I was honestly thinking of an answer choice having something along the lines of mediocre form of government being better than inherently best or worst. I had no clue about the answer choices. Can someone help me?
 
timmydoeslsat
Thanks Received: 887
Atticus Finch
Atticus Finch
 
Posts: 1136
Joined: June 20th, 2011
 
This post thanked 5 times.
 
trophy
Most Thanked
trophy
First Responder
 

Re: Q13 - Philosopher: Some of the most

by timmydoeslsat Thu Sep 15, 2011 11:27 pm

I will help!

We have a stimulus that states that the best and worst of government are those that have power concentrated in the hands of a few. It also states with since, which is a premise, that democracy is a consistently mediocre form of government.

The argument concludes that democracy is a better choice than rule by the few.

So we need a principle to justify the idea that democracy is a better choice than the rule by the few.

It is an invalid argument as of now. Just because democracy is a consistently mediocre form of government, why is that a better choice than the rule by few? I mean with the rule by the few, you do have the opportunity for the best of government. Although, you do have the opportunity for the worst as well.

I was sensing a principle that would state that we want to avoid a type of rule that could place us in a situation of the worst of government.


Answer choices:

A) Biconditional statement.

Society should adopt a democratic form of government <---> Most members of the society prefer a democratic form of government

Each side of the arrow needs the other.

In this case, it does not matter. The only direction of the arrow we can use would be the one that would have:

Most members of the society prefer a democratic form of government ---> Society should adopt a democratic form of government

This is because we want to be able to end with the variable we are trying to conclude. And the sufficient condition, in this case, is not triggered. It is not touched upon in the stimulus. Plus, even if we did reach this necessary condition, it does not touch upon the conclusion reached in the argument of democracy being the better choice.


B) This is our answer.

In choosing a form of government, it is better for a society to avoid the inherently worst than to seek to attain the best.

This principle lets us know that it is better to avoid the inherently worse than to attain the best.

So we know that rule by the few will let you have the best and worst of government.

Democracy is mediocre, meaning it is not AS BAD as rule by the few. Due to that fact, we know that democracy is better.



Think about it this way. What if this principle instead said:


In choosing a form of government, it is better for a society to seek to attain the best than to avoid the inherently worst.


That would support the idea of "rule by the few" being a better choice than democracy.


C) Do not know about likelihood.

D) Does not support which is a better choice.

E) Not touched upon in stimulus.